DON'T WANT TO MISS A THING?

Certification Exam Passing Tips

Latest exam news and discount info

Curated and up-to-date by our experts

Yes, send me the newsletter

Latest Juniper JNCIP-SP JN0-664 Exam Questions for Comprehensive Preparation

Preparing for the Juniper JN0-664 exam can be a daunting task, but with the right study materials and exam resources, you can increase your chances of passing successfully. SPOTO's Juniper JN0-664 exam questions and answers provide a comprehensive set of test questions that cover all the topics and objectives of the actual exam. These study materials are meticulously crafted by industry experts, ensuring their accuracy and relevance. With SPOTO's mock exams, you can simulate the real exam environment and gauge your readiness. Additionally, their exam preparation resources offer valuable insights and tips to help you effectively prepare for the exam. By leveraging these exam resources, you can confidently approach the Juniper JN0-664 certification and enhance your career prospects in the networking industry. JNCIP-SP, the professional-level certification in this track, is designed for networking professionals with advanced knowledge of routing and switching implementations in Junos. The written exam verifies your basic understanding of advanced routing technologies and related platform configuration and troubleshooting skills.
Take other online exams

Question #1
08. How does BGP Layer 2 VPN establish Layer 2 connectivity between customer sites?
A. SCPfield
B. Pprecedencefield
C. OSfield
D. XPfield
View answer
Correct Answer: B
Question #2
Click the Exhibit button.You are troubleshooting the connection between AS 64496 and AS 64497 and notice that only one of the paths is being used for traffic forwarding.Referring to the exhibit, which three actions will ensure that R1 is configured properly for load balancing BGP routes? (Choose three.)
A. erify that the routing table on R1 has BGP routes for 203
B. erify that the multipath option is configured under protocols bgp on both R2 and R3
C. erify that there is a load balancing export policy under routing-options for the received BGP routes on R1
D. erify that the multipath option is configured under protocols bgp on R1
E. erify that an import load balancing policy exists under protocols bgp for the received BGP routes on R1
View answer
Correct Answer: ABD
Question #3
Click the Exhibit button.Referring to the exhibit. PE-1 and PE-2 are getting route updates for VPN-B when neither of them service that VPN.Which two actions would optimize this process? (Choose two.)
A. Configure the resolution rib bgp
B. Configure the family route-target statement on the RR
C. Configure the resolution rib bgp
D. Configure the family route-target statement on the PEs
View answer
Correct Answer: BD
Question #4
ExhibitYou must ensure that the VPN backbone is preferred over the back door intra-area link as long as the VPN is available. Referring to the exhibit, which action will accomplish this task?
A. onfigure an import routing policy on the CE routers that rejects OSPF routes learned on the backup intra-area link
B. nable OSPF traffic-engineering
C. onfigure the OSPF metric on the backup intra-area link that is higher than the L3VPN link
D. reate an OSPF sham link between the PE routers
View answer
Correct Answer: D
Question #5
Click the Exhibit button.You want to implement the BGP Generalized TTL Security Mechanism (GTSM) on the network.Which three statements are correct in this scenario? (Choose three.)
A. BGP GTSM requires a TTL of 1 to be configured between neighbors
B. You can implement BGP GTSM between R2, R3, and R4
C. You can implement BGP GTSM between R2 and R1
D. BGP GTSM requires a firewall filter to discard packets with incorrect TTL
E. BGP GTSM requires a TTL of 255 to be configured between neighbors
View answer
Correct Answer: BCE
Question #6
Click the Exhibit button.Which two statements are correct about the class-of-service configuration shown in the exhibit? (Choose two.)
A. Incoming traffic will be classified using the default classifier
B. The best-effort queue can never transmit more than 40% of the total bandwidth on the ge-0/0/0 interface, even if that bandwidth is available
C. Incoming traffic will not be classified because no classifier exists in the configuration
D. The best-effort queue can transmit more than 40% of the total bandwidth on the ge-0/0/0 interface, if no other queue is using that bandwidth
View answer
Correct Answer: AD
Question #7
03. How are packets classified in Junos OS CoS?
A. asedontheirsourceMACaddresse
B. asedontheirdestinationMACaddresse
C. asedontheirsourceanddestinationIPaddresse
D. asedontheirprotocolandportnumbe
View answer
Correct Answer: C
Question #8
09. Which CoS field is used to specify the desired loss priority of a packet?
A. o-exp
B. o-advertise
C. ocal-AS
D. nterne
View answer
Correct Answer: D
Question #9
Exhibituser@Rl show configuration interpolated-profile{ interpolate { fill-level [ 50 75 drop--probability [ > }class-of-service drop-profiles];20 60 ];Which two statements are correct about the class-of-service configuration shown in the exhibit? (Choose two.)
A. onfigure the explicit-null statement on PE-1
B. onfigure the explicit-null statement on PE-2
C. onfigure VPN prefix mapping for the PE-1_to_PE-2 LSP
D. et a static CoS value for the PE-1_to_PE-2 LSP
View answer
Correct Answer: BC
Question #10
ExhibitR1 and R8 are not receiving each other's routesReferring to the exhibit, what are three configuration commands that would solve this problem? (Choose three.)
A. he packet will be discarded
B. he packet will be processed by the BA classifier first, then the MF classifier
C. he packet will be forwarded with no classification changes
D. he packet will be processed by the MF classifier first, then the BA classifier
View answer
Correct Answer: BDE
Question #11
You are asked to protect your company's customers from amplification attacks.In this scenario, what is Juniper's recommended protection method?
A. ASN prepending
B. BGP FlowSpec
C. destination-based Remote Triggered Black Hole
D. unicast Reverse Path Forwarding
View answer
Correct Answer: B
Question #12
You are responding to an RFP for a new MPLS VPN implementation. The solution must use LDP for signaling and support Layer 2 connectivity without using BGP The solution must be scalable and support multiple VPN connections over a single MPLS LSP The customer wants to maintain all routing for their Private network.In this scenario, which solution do you propose?
A. hange the level l disable parameter under the R1 protocols isis interface lo0
B. emove the level i disable parameter under the R2 protocols isis interface loo
C. hange the level 1 disable parameter under the R2 protocols isis interface ge-1/2/3
D. dd IP addresses to the interface ge-l/2/3 unit 0 family iso hierarchy on both R1 and R2
View answer
Correct Answer: C
Question #13
In IS-IS. which two statements are correct about the designated intermediate system (DIS) on a multi-access network segment? (Choose two.)
A. On the multi-access network, each router forms an adjacency to every other router on the segment
B. On the multi-access network, each router only forms an adjacency to the DIS
C. A router with a priority of 1 wins the DIS election over a router with a priority of 10
D. A router with a priority of 10 wins the DIS election over a router with a priority of 1
View answer
Correct Answer: AD
Question #14
ExhibitThe environment is using BGP All devices are in the same AS with reachability redundancy Referring to the exhibit, which statement is correct?
A. R1 is peered to Client2 and RR2
B. R2 is in an OpenConfirm State until RR1 becomes unreachable
C. lient1 is peered to Client2 and Client3
D. eering is dynamically discovered between all devices
View answer
Correct Answer: A
Question #15
Which origin code is preferred by BGP?
A. Internal
B. External
C. Incomplete
D. Null
View answer
Correct Answer: A
Question #16
Which two statements are correct about a sham link? (Choose two.)
A. nternal
B. xternal
C. ncomplete
D. ull
View answer
Correct Answer: AC
Question #17
You are configuring a BGP signaled Layer 2 VPN across your MPLS enabled core network. Your PE-2 device connects to two sites within the s VPNIn this scenario, which statement is correct?
A. y default on PE-2, the site's local ID is automatically assigned a value of 0 and must be configured to match the total number of attached sites
B. ou must create a unique Layer 2 VPN routing instance for each site on the PE-2 device
C. ou must use separate physical interfaces to connect PE-2 to each site
D. y default on PE-2, the remote site IDs are automatically assigned based on the order that you add the interfaces to the site configuration
View answer
Correct Answer: D
Question #18
Your network is receiving the 203.0.113.0/24 network using EBGP from AS 64500 and AS 64501. Both of these advertisements have identical local-preference values, AS-path lengths, and BGP origin codes. You want to influence the way your AS sends traffic to the 203.0.113.0/24 network.In this scenario, which attribute would you consider next when selecting the best path?
A. router ID
B. MED value
C. peer IP address
D. IGP metric
View answer
Correct Answer: B
Question #19
You are using a Layer 3 VPN to connect two customer sites. The VPN routes for the customer networks appear as hidden in the bgp.l3vpn.0 routing table on the PE routers.What is causing this problem?
A. The routes use overlapping IP addresses
B. There is a routing loop in the service provider backbone
C. There is not an established MPLS LSP between the two PE routers
D. Route targets are not configured
View answer
Correct Answer: C
Question #20
You have an L2VPN connecting two CEs across a provider network. The CEs and provider network are configured with the default MTU setting. You use the ping command from one CE to the other CE with a size of 1500 bytes.In this scenario, which statement is correct when using the ping command?
A. You expect an ICMP message too long error
B. You expect an echo reply
C. You expect a silent discard
D. You expect the ping results to be fragmented
View answer
Correct Answer: B

View Answers after Submission

Please submit your email and WhatsApp to get the answers of questions.

Note: Please make sure your email ID and Whatsapp are valid so that you can get the correct exam results.

Email:
Whatsapp/phone number: